how do you write in numerals

Answers

Answer 1

Answer:

Basic numbersNumbers up to nine should always be written in words, anything higher than nine can be written in numerals. Alternatively, some guides suggest that if you can write the number in two words or fewer then use words rather than number.

hope it's helpful for u !!

stay safe ...

Answer 2

Step-by-step explanation:

1 : a conventional symbol that represents a number. 2 numerals plural : numbers that designate by year a school or college class and that are awarded for distinction in an extracurricular activity. Other Words from numeral Synonyms Example Sentences Learn More About numeral.

A numeral is a symbol or name that stands for a number. Examples: 3, 49 and twelve are all numerals. So the number is an idea, the numeral is how we write it.

hope it helps.stay safe healthy and happy...

Related Questions

WILL GIVE BRAINLIEST!!!
Write as a polynomial: 14b + 1 - 6(2 - 11b)

Answers

Answer:

80b-11

Step-by-step explanation:

14b + 1 - 6(2 - 11b)

Distribute

14b+1-12+66b

Combine like terms

80b-11

Answer:

80b - 11

Step-by-step explanation:

what is the problem ?

just multiply it out and combine terms.

14b + 1 - 6(2 - 11b) = 14b + 1 - 12 + 66b = 80b - 11

2(-x-4)+3=-7x+5+5x

Pls help!!!!!!!!

Answers

4 I hope this helps you

1 Simplify

7
x
+
5
+
5
x
−7x+5+5x to

2
x
+
5
−2x+5.
2
(

x

4
)
+
3
=

2
x
+
5
2(−x−4)+3=−2x+5

2 Expand.

2
x

8
+
3
=

2
x
+
5
−2x−8+3=−2x+5

3 Simplify

2
x

8
+
3
−2x−8+3 to

2
x

5
−2x−5.

2
x

5
=

2
x
+
5
−2x−5=−2x+5

4 Cancel

2
x
−2x on both sides.

5
=
5
−5=5

5 Since

5
=
5
−5=5 is false, there is no solution.
No Solution

Find the distance between the points (-5, -4) and (3, 1).

On a coordinate plane, points are at (3, 1), (negative 5, negative 4).

Answers

Step-by-step explanation:

it will help u

Find the Taylor series for f(x) centered at the given value of a. (Assume that f has a power series expansion. Do not show that Rn(x)→0 . f(x)=lnx, a=

Answers

Answer:

Here we just want to find the Taylor series for f(x) = ln(x), centered at the value of a (which we do not know).

Remember that the general Taylor expansion is:

[tex]f(x) = f(a) + f'(a)*(x - a) + \frac{1}{2!}*f''(a)(x -a)^2 + ...[/tex]

for our function we have:

f'(x) =  1/x

f''(x) = -1/x^2

f'''(x) =  (1/2)*(1/x^3)

this is enough, now just let's write the series:

[tex]f(x) = ln(a) + \frac{1}{a} *(x - a) - \frac{1}{2!} *\frac{1}{a^2} *(x - a)^2 + \frac{1}{3!} *\frac{1}{2*a^3} *(x - a)^3 + ....[/tex]

This is the Taylor series to 3rd degree, you just need to change the value of a for the required value.

Linda found that the cost to get a swimming pool installed in her backyard is a linear function of the pool's area. A swimming pool with an area of 1,000 square feet can be installed for $50,000, whereas the installation of an 800 square foot swimming pool costs $35,000. Select the correct graph that models the given relationship.

Answers

Answer:

$35,000

Step-by-step explanation:

if $50,000 is to install an area of 1,000 square feet swimming pool and $35,000 can be used to install an 800 square foot swimming pool I think the best graph model is 800 square feet for $35,000 for a cost cut of $15,000 is a good bargain

CAN SOMEONE HELP ME ON ANALYZING DOT PLOTS!!!

Answers

Answer:

yes

Step-by-step explanation:

but I can't see them here

John's age 4 years ago, if he will be y years old in 5 years

Answers

9514 1404 393

Answer:

  y -9

Step-by-step explanation:

From 4 years ago until 5 years from now, John will age 9 years. That is, his age 4 years ago is 9 years less than it will be in 5 years.

John's age 4 years ago is y-9 years.

The function c(r)=2r+12.5 represents the cost c, in dollars, of riding r rides
at a carnival. How much does it cost to get into the carnival? *
1 point
A.$2
B. $12.50
C. $14.50
D.r

Answers

so basically i think it is c or maybe b do inie meni myni mo

In 2013, the Public Religion Research Institute conducted a survey of 1,033 adults, 18 years of age or older, in the continental United States. One of the questions on their survey was as follows:

Answers

Answer:

Probability[Number of people from church] = 0.26 (Approx.)

Step-by-step explanation:

Given:

Total number of adult in survey = 1,033

Missing information:

Number of people from church = 269

Find:

Probability[Number of people from church]

Computation:

Probability of an event = Number of favourable outcomes / Number of total outcomes

Probability[Number of people from church] = Number of people from church / Total number of adult in survey

Probability[Number of people from church] = 269 / 1,033

Probability[Number of people from church] = 0.2604

Probability[Number of people from church] = 0.26 (Approx.)





Which expression has a value of 15 when it equals
2
49-57
3--5
61-28
28
19

Answers

Answer:

it is 61-28 but I not sure u can scan for any application to make sure u get it ur answer thx for

Need the answers from a - e

Answers

Answer:

10

Step-by-step explanation:

Sorry. I needed to answer this question to get access.

There are 92 students enrolled in an French course and 248 students enrolled in a Spanish course. Construct a ratio comparing students enrolled in a French course to students enrolled in a Spanish course. Write your answer as a decimal, rounded to the thousandths place.

Answers

Answer:

0.371

Step-by-step explanation:

The ratio comparing students enrolled in a French course to students enrolled in a Spanish course rounded to the thousandths place is 0.371.

What is the ratio?

A ratio indicates how many times one number contains another. If a and b are to objects then ratio of a to the b is given as a : b.

Now it is given that,

Students enrolled in a French course = 92

Students enrolled in a Spanish course = 248

So, Ratio comparing students enrolled in a French course to students enrolled in a Spanish = Students enrolled in a French course / Students enrolled in a Spanish course

Ratio comparing students enrolled in a French course to students enrolled in a Spanish = 92/248

Ratio comparing students enrolled in a French course to students enrolled in a Spanish = 0.370967

To rounded to the thousandths place, the digit at the thousandth place is 0 and right to it is 9 which is greater than 5 so round up the place value at thousandths place.

Ratio comparing students enrolled in a French course to students enrolled in a Spanish = 0.371

Thus, the ratio comparing students enrolled in a French course to students enrolled in a Spanish course rounded to the thousandths place is 0.371.

To learn more about ratio:

https://brainly.com/question/1504221

#SPJ2

In your office desk drawer you have 10 different flavors of fruit leather. How many distinct flavor groupings can you make with your fruit leather stash?​

Answers

100

You have 10 different flavors. Just multiply the ten with another ten.

Children arrive at a house to do Halloween trick-or- treating according to a Poisson process at the unlucky rate of 13/hour. What is the probability that the time between the 15th and 16th arrivals will be more than 4 minutes ? (Hint: Think exponential.)
a) e e-2 = 0.1353
b) e-13/15 = 0.4204
c) e-1 = 0.3679
d) 1-2-1 = 0.6321

Answers

Answer:

0.4204 probability, option b.

Step-by-step explanation:

Exponential distribution:

The exponential probability distribution, with mean m, is described by the following equation:

[tex]f(x) = \mu e^{-\mu x}[/tex]

In which [tex]\mu = \frac{1}{m}[/tex] is the decay parameter.

The probability that x is lower or equal to a is given by:

[tex]P(X \leq x) = \int\limits^a_0 {f(x)} \, dx[/tex]

Which has the following solution:

[tex]P(X \leq x) = 1 - e^{-\mu x}[/tex]

The probability of finding a value higher than x is:

[tex]P(X > x) = 1 - P(X \leq x) = 1 - (1 - e^{-\mu x}) = e^{-\mu x}[/tex]

Children arrive at a house to do Halloween trick-or- treating according to a Poisson process at the unlucky rate of 13/hour

13 arrivals during an hour, which means that the mean time between arrivals, in minutes is of [tex]\mu = \frac{13}{60} = 0.2167[/tex]

What is the probability that the time between the 15th and 16th arrivals will be more than 4 minutes ?

This is P(X > 4). So

[tex]P(X > 4) = e^{-0.2167*4} = 0.4204[/tex]

So the correct answer is given by option b.

2/9 divided by 5/6
help pleaseee​

Answers

Hey there!

[tex]\mathsf{\dfrac{2}{9}\div\dfrac{5}{6}}[/tex]

[tex]\mathsf{= \dfrac{2\times6}{9\times5}}[/tex]

[tex]\mathsf{2\times 6 = \bf 12}[/tex]

[tex]\mathsf{9\times5 = \bf 45}[/tex]

[tex]\boxed{\mathsf{=\bf \dfrac{12}{45}}}[/tex]

[tex]\large\textsf{BOTH NUMBERS has the Greatest Common Factor (GCF) of 3}[/tex]

[tex]\mathsf{= \dfrac{12\div3}{45\div3}}[/tex]

[tex]\mathsf{12\div3=\bf 4}[/tex]

[tex]\mathsf{45\div3=\bf 15}[/tex]

[tex]\boxed{\mathsf{=\bf \dfrac{4}{15}}}[/tex]

[tex]\boxed{\boxed{\large\textsf{Answer: }\mathsf{\bf \dfrac{4}{15}}}}\huge\checkmark[/tex]

[tex]\large\textsf{Good luck on your assignment and enjoy your day!}\\\\\\~\frak{Amphitrite1040:)}}[/tex]

In order to win a prize, Heather randomly draws two balls from a basket of 40. There are 25 blue balls, and the rest are green balls. Of the blue balls, 12% are winning balls. Of the green balls, 20% are winning balls. Calculate the expected number of winning balls that Heather draws.

Answers

Answer:

The expected number of winning balls that Heather draws is 0.3.

Step-by-step explanation:

The balls are chosen without replacement, which means that the hypergeometric distribution is used to solve this question.

Hypergeometric distribution:

The probability of x successes is given by the following formula:

[tex]P(X = x) = h(x,N,n,k) = \frac{C_{k,x}*C_{N-k,n-x}}{C_{N,n}}[/tex]

In which:

x is the number of successes.

N is the size of the population.

n is the size of the sample.

k is the total number of desired outcomes.

Combinations formula:

[tex]C_{n,x}[/tex] is the number of different combinations of x objects from a set of n elements, given by the following formula.

[tex]C_{n,x} = \frac{n!}{x!(n-x)!}[/tex]

Expected value of the hypergeometric distribution:

The expected value is given by:

[tex]E(X) = \frac{nk}{N}[/tex]

Expected number of blue and green balls:

40 balls, which means that [tex]N = 40[/tex]

2 are chosen, which means that [tex]n = 2[/tex]

25 are blue, which means that [tex]k = 25[/tex]

So

[tex]E(X) = \frac{nk}{N} = \frac{25(2)}{40} = 1.25[/tex]

1.25 balls are expected to be blue and 2 - 1.25 = 0.75 green.

Of the blue balls, 12% are winning.

Of the green balls, 20% are winning.

Calculate the expected number of winning balls that Heather draws.

[tex]E_w = 1.25*0.12 + 0.75*0.2 = 0.3[/tex]

The expected number of winning balls that Heather draws is 0.3.

Solve the system, or show that it has no solution. (If there is no solution, enter NO SOLUTION. If there are an infinite number of solutions, enter the general solution in terms of x, where x is any real number.)
20x − 80y = 100
−14x + 56y = −70
(x, y) =

Answers

Answer:

The system has an infinite set of solutions [tex](x,y) = (x, \frac{x-5}{4})[/tex]

Step-by-step explanation:

From the first equation:

[tex]20x - 80y = 100[/tex]

[tex]20x = 100 + 80y[/tex]

[tex]x = \frac{100 + 80y}{20}[/tex]

[tex]x = 5 + 4y[/tex]

Replacing on the second equation:

[tex]-14x + 56y = -70[/tex]

[tex]-14(5 + 4y) + 56y = -70[/tex]

[tex]-70 - 56y + 56y = -70[/tex]

[tex]0 = 0[/tex]

This means that the system has an infinite number of solutions, considering:

[tex]x = 5 + 4y[/tex]

[tex]4y = x - 5[/tex]

[tex]y = \frac{x - 5}{4}[/tex]

The system has an infinite set of solutions [tex](x,y) = (x, \frac{x-5}{4})[/tex]

is this right? PLEASE HELP ILL MARK

Answers

Answer:

yeah u r correct...

hope it helps ..stay safe healthy and happy.

The solution of this equation has an error. Which of the following steps has the error? 18 − (3x + 5) = 8
Step 1: 18 − 3x + 5 = 8
Step 2: -3x + 23 = 8
Step 3: -3x = -15
Step 4: x = 5



Step 1 Step 2 Step 3 Step 4. ?

Answers

Answer:

Step 1

Because the number in front of the bracket is 1 and it is also affected by the negative sign(-),5 is supposed to be negative not positive because (negative by positive is negative)

And since the first step has an error in it,the remaining steps would also be wrong.

Question 2
A force F=5i+3j-2k is applied to move a block of cement from A(0,1,1) to B(4.-1,3).
Determine the work done by the force.

Answers

The work is simply the dot product of the force and displacement (which I assume are given in Newtons and meters, respectively):

W = F • d

W = (5i + 3j - 2k) N • ((4i - j + 3k) m - (j + k) m)

W = (5i + 3j - 2k) • (4i - 2j + 2k) Nm

W = (20 - 6 - 4) Nm

W = 10 J

solve the inequality 4t^2 ≤ 9t-2 please show steps and interval notation. thank you!​

Answers

Answer:

[0.25, 2]

Step-by-step explanation:

We have

4t² ≤ 9t-2

subtract 9t-2 from both sides to make this a quadratic

4t²-9t+2 ≤ 0

To solve this, we can solve for 4t²-9t+2=0 and do some guess and check to find which values result in the function being less than 0.

4t²-9t+2=0

We can see that -8 and -1 add up to -9, the coefficient of t, and 4 (the coefficient of t²) and 2 multiply to 8, which is also equal to -8 * -1. Therefore, we can write this as

4t²-8t-t+2=0

4t(t-2)-1(t-2)=0

(4t-1)(t-2)=0

Our zeros are thus t=2 and t = 1/4. Using these zeros, we can set up three zones: t < 1/4, 1/4<t<2, and t>2. We can take one random value from each of these zones and see if it fits the criteria of

4t²-9t+2 ≤ 0

For t<1/4, we can plug in 0. 4(0)²-9(0) + 2 = 2 >0 , so this is not correct

For 1/4<t<2, we can plug 1 in. 4(1)²-9(1) +2 = -3 <0, so this is correct

For t > 2, we can plug 5 in. 4(5)²-9(5) + 2 = 57 > 0, so this is not correct.

Therefore, for 4t^2 ≤ 9t-2 , which can also be written as 4t²-9t+2 ≤ 0, when t is between 1/4 and 2, the inequality is correct. Furthermore, as the sides are equal when t= 1/4 and t=2, this can be written as [0.25, 2]

On weekend nights, a large urban hospital has an average of 4.8 emergency arrivals per hour. Let X be the number of arrivals per hour on a weekend night at this hospital. Assume that successive arrivals are random and independent. What is the probability P(X < 3)?

Answers

Answer:

P(X < 3) = 0.14254

Step-by-step explanation:

We have only the mean, which means that the Poisson distribution is used to solve this question.

Poisson distribution:

In a Poisson distribution, the probability that X represents the number of successes of a random variable is given by the following formula:

[tex]P(X = x) = \frac{e^{-\mu}*\mu^{x}}{(x)!}[/tex]

In which

x is the number of sucesses

e = 2.71828 is the Euler number

[tex]\mu[/tex] is the mean in the given interval.

On weekend nights, a large urban hospital has an average of 4.8 emergency arrivals per hour.

This means that [tex]\mu = 4.8[/tex]

What is the probability P(X < 3)?

[tex]P(X < 3) = P(X = 0) + P(X = 1) + P(X = 2)[/tex]

So

[tex]P(X = x) = \frac{e^{-\mu}*\mu^{x}}{(x)!}[/tex]

[tex]P(X = 0) = \frac{e^{-4.8}*4.8^{0}}{(0)!} = 0.00823[/tex]

[tex]P(X = 1) = \frac{e^{-4.8}*4.8^{1}}{(1)!} = 0.03950[/tex]

[tex]P(X = 2) = \frac{e^{-4.8}*4.8^{2}}{(2)!} = 0.09481[/tex]

So

[tex]P(X < 3) = P(X = 0) + P(X = 1) + P(X = 2) = 0.00823 + 0.03950 + 0.09481 = 0.14254[/tex]

P(X < 3) = 0.14254

need help now!!! Please and thanks ​

Answers

Answer:

the answer of r is 8 i hope it will help

Which choice correctly shows the line y = -x?
А
B
NOW
-
1 2 3 4
NH
-4 -3 -2 -1 1 2 3 4
UN
С
2
1 2 3 4
-4-3-2/4 1 2 3 4
-4 -3 -2 -3
NA
2
At
2

Answers

Answer:

The answer is A

Step-by-step explanation:

Hope this helps

una fuerza constante F de magnitud igual a 3lb se aplica al bloque que se muestra en la figura. F tiene la misma dirección que el vector a= 3i + 4j. determine el trabajo realizado en la dirección de movimiento si el bloque se mueve de P1 (3, 1) a P2 (9, 3). Suponga que la distancia se mide en pies.

Answers

Uh is that Spanish? Bc I can’t speak Spanish

Identify the domain of the table of values shown

Answers

Answer:

{-6,0,2,4}

Step-by-step explanation:

the graph function f(x) is illustrated in figure below (-2,1) ,(-1,2) ,(1,2) ,(2,3) .Use the transformation techniques to graph the following functions

a) y=f(x)-2
b) y=f(-x)

Answers

Answer:

a) y = f(x) - 2 (x, y) ⇒ (x, y - 2)b) y = f(-x) (x, y) ⇒ (-x, y)

a) y=f(x)-2

(-2, 1) → (-2, 1 - 2) = (-2, -1)(-1, 2) → (-1, 2 - 2) = (-1, 0)(1, 2) → (1, 2 - 2) = (1, 0)(2, 3) → (2, 3 - 2) = (2, 1)

b) y=f(-x)

(-2, 1) → (-(-2), 1) = (2, 1)(-1, 2) → (-(-1), 2) = (1, 2)(1, 2) → (-1, 2)(2, 3) → (-2, 3)

Find the value of x pls help

Answers

9514 1404 393

Answer:

  x = 36°

Step-by-step explanation:

The exterior angle is equal to the sum of the remote interior angles. A linear pair is supplementary. So, you can find x either of two ways:

  2x = x + (180 -4x)   ⇒   5x = 180   ⇒   x = 36

Or ..

  4x = x + (180 -2x)   ⇒   5x = 180   ⇒   x = 36

The value of x is 36°.

1) What is the opposite of adding 5?
2) What is the opposite of subtracting 20?
3) What is the opposite of multiplying by 1/2?
4) What is the opposite of dividing by 10?

Answers

Answer:

1) subtracting 5

2) adding 20

3) dividing by 2 (multiplying by 1/2)

4) multiplying by 1/10 (dividing by 10)

Step-by-step explanation:

There are four main operations in math: adding, subtracting, multiplying, and dividing. Each of the operations has an opposite. Adding and subtracting are opposites and multiplying and dividing are opposites. This means that subtracting can undo adding and vice versa; additionally, dividing can undo multiplying or vice versa. So, to find the opposite of something switch the operation to the opposite and keep the number. However, it is important to note that with multiplying and dividing you can also find the opposite by keeping the operation while changing the number to the reciprocal.

A statistics professor plans classes so carefully that the lengths of her classes are uniformly distributed between 47.0 and 57.0 minutes. Find the probability that a given class period runs between 51.25 and 51.5 minutes.

Answers

Answer:

0.025 = 2.5% probability that a given class period runs between 51.25 and 51.5 minutes.

Step-by-step explanation:

Uniform probability distribution:

An uniform distribution has two bounds, a and b.

The probability of finding a value between c and d is:

[tex]P(c \leq X \leq d) = \frac{d - c}{b - a}[/tex]

Uniformly distributed between 47.0 and 57.0 minutes.

This means that [tex]a = 47, b = 57[/tex]

Find the probability that a given class period runs between 51.25 and 51.5 minutes.

[tex]P(c \leq X \leq d) = \frac{51.5 - 51.25}{57 - 47} = 0.025[/tex]

0.025 = 2.5% probability that a given class period runs between 51.25 and 51.5 minutes.

Other Questions
tipos de escalas sismicas What is an html skeleton? If you know what it is can you write it for me? Is it a, b, c or d? Please help Look at pic for question and answer choices. Ali while driving to school computes the average speed for his trip to be 20km/hr on his return trip along the same route. There is less traffic and average speed is 30km/hr. What is the average speed for Ali's trip? How do people engage in Hip Hop? Which southern African country was ruled by the dictator Robert Mugabeafter he led a successful rebellion against its earlier white-controlledgovernment? The sum of the digits in a 2 digit number is 5. If the number is subtracted by 9 then the digits will be reversed. Find the number. If the tens digit is x then what is the equation? can someone please help me.. what is the meaning of compiler Kenneth ran a marathon (26.2) miles) in 5.5 hours. What was Kenneths average speed? (Round your answer to the nearest tenth.) Square Footage Frequency0-499 5500-999 171000-1499 361500-1999 1152000-2499 1252500-2999 813000-3499 473500-3999 454000-4499 224500-4999 7The accompanying frequency distribution represents the square footage of a random sample of 500 houses that are owner occupied year round. Approximate the mean and standard deviation square footage. You are analyzing ABC Company, a computer manufacturer. You notice that inventory turnover this year is significantly lower than in prior years. You also notice that accounts receivable turnover is significantly lower this year when compared to previous years. Provide three explanations that would be consistent with your observation for inventory turnover and include an explanation of whether these would be of concern to you, as well as what the effect might be on the next period's financial results. In addition, provide three explanations that would be consistent with your observation of the accounts receivable turnover, and explain whether these would be of concern to you. When people are asked to recall a list of words they had earlier memorized, they often substitute common synonyms for some of the words on the original list. This best illustrates the effects of how is mercutio related to the play Which occupation is an example of a civil service position?A. doctorB. police officerC. lawyerD. web designer GIVE AN EXPLANATION PLEASE! A BRAINLEEST IF TWO OR MORE PEOPLE HAVE ANSWERED IT. (FIRST COME FIRST SERVE BASES + ANSWER HAS TO BE CORRECT)Lily bought a pair of gloves and a shirt.The gloves cost 4She sold the gloves and the shirt for a total of 48She made 100% profit on the cost of the gloves.20% profit on the total cost.Work out her percentage profit on the cost of the skirt.Answer 1 decimal place. write a essay about the great chicago fire Computer data that is suitable for text PLEASE HELP TIMED. Verify which of the following are identities.